Two squared equals to three

Logic Level 3

T W O 2 = T H R E E \overline{\mathsf{TWO}}^2 = \overline{\mathsf{THREE}}

Solve the cryptarithm above and find the value of T + W + H + R \mathsf{T} + \mathsf{W} + \mathsf{H} + \mathsf{R} .

Details

  • T , W , H , R T,W,H,R are each distinct integers { 0 , 9 } \in \{0,\ldots9\} .
This problem is from my set Numbers in Disguise - Cryptarithms .


The answer is 13.

This section requires Javascript.
You are seeing this because something didn't load right. We suggest you, (a) try refreshing the page, (b) enabling javascript if it is disabled on your browser and, finally, (c) loading the non-javascript version of this page . We're sorry about the hassle.

4 solutions

T W O 2 = T H R E E \overline { TWO } ^{ 2 }\quad =\quad \overline { THREE }

For the above cryptarithm to be true, T can only be 1. But if W is 4 or larger, the above cryptarithm is also untrue. Therefore, W can either be 2 or 3. Let W = 2 and trying out O from 0, 3 to 9., there is no solution. For W =3, it is found that O = 8 gives the solution.

13 8 2 = 19044 138^{2} = 19044

Therefore, T=1, W=3, H=9 and R=0 and

T + W + H + R = 1 + 3 + 9 + 0 = 13 T+W+H+R = 1+3+9+0 = \boxed{13}

Sir, can you explain your conjectures? I am unable to comprehend them.

Raven Herd - 6 years, 3 months ago
Tom Van Lier
Oct 21, 2015

There is something wrong with the statement of the problem. It says that T,W,H and R are distinct integers. Not what E and O are.

Therefor, 140² = 19600 is also a right solution as is 130² = 16900. In both cases T,W, H and R are distinct integers. O and E aren't but that isn't required.

Ameya Salankar
May 1, 2014

The cryptarithm is:

13 8 2 = 19044 138^2 = 19044

& the value of T + W + H + R = 1 + 3 + 9 + 0 = 13 \mathsf{T} + \mathsf{W} + \mathsf{H} + \mathsf{R} = 1 + 3 + 9 + 0 = \boxed{13} .

I solve it for T W O = 138 \overline {TWO} =138 but why T W O 2 = 11 2 2 = 12544 = T H R E E \overline {TWO^{2}}=112^{2}=12544=\overline {THREE} is incorrect? There wasn't any condition says H O \overline {H} \neq \overline {O}

Mahdi Al-kawaz - 7 years, 1 month ago

Log in to reply

@Mahdi Al-kawaz , in a cryptarithm, a number is denoted by a fixed letter which never changes in a given problem.

This would mean that H O \mathsf{H} \neq \mathsf{O} .

Ameya Salankar - 7 years, 1 month ago

Neither can W=T

Guiseppi Butel - 7 years, 1 month ago

How to go about it? I used cpp. Is there a better solution?

Himanshu Arora - 7 years ago

still wondering why you asked for T + W + H + R \text{T}+\text{W}+\text{H}+\text{R} instead of T + W + O + H + R + E \text{T}+\text{W}+\text{O}+\text{H}+\text{R}+\text{E} . I got one of my tries incorrect because of that :/

mathh mathh - 6 years, 10 months ago

THREE has odd number of digits, and last digits of TWO and THREE are same. T = 1. L e t E u b e u n i t d g i t . E t t h e t e n t h . Since T=1, it recives no carry. So W=1, 2, or 3. We can see that the following conditions must hold good. No two symbols can represent the same digit. So W=2,3 and O E u . 0,1,5,6 are eliminated from O and E u . E u c a n n o t b e 2 , 3 , 7 , o r 8. Because no square of O will end in these. E u c a n o n l y b e 4 , o r 9. F o r E u to be 4, or 9..... O have to be 2 or 8, OR 3 or 7 . S i n c e E u = E t , i f E u = 9 , O= 3 or 7, an odd digit, so there is no carry or an even carry. Add to the carry the contribution of W 2 t o E t . This is always even. So this makes E t a n e v e n d i g i t ! E i s o n l y e v e n . So only option remaining O=2 or 8 and W= 2 or 3. We can not have W=O=2. So only options remaining are 128, 132, and 138. So we see that 13 8 2 = 19044.. h i t s t h e t a r g e t ! ! ! ! T + W + H + R = 1 + 3 + 9 + 0 = 13 \text{ THREE has odd number of digits, and last digits of TWO and THREE }\\\text{are same. } ~~~\therefore~~~~T=1.~~~~~~~~~~~~~Let ~E_u ~ be ~ unit~ dgit.~~~~~~E_t~the~tenth. \\ \text{Since T=1, it recives no carry. So W=1, 2, or 3. We can see that the } \\\text{following conditions must hold good.} \\ \text{No two symbols can represent the same digit. So W=2,3 and O } \ne E_u. \\\implies\text{0,1,5,6 are eliminated from O and }E_u.~\therefore E_u ~ can~ not~ be~ 2,3,7, or~ 8.\\ \text{ Because no square of O will end in these.} ~E_u~ can~ only ~be~ 4,~ or~ 9. \\For ~~E_u \text{ to be 4, or 9..... O have to be 2 or 8, OR 3 or 7 . } \\Since~E_u=E_t, ~if~E_u=9, ~\text{ O= 3 or 7, an odd digit, so there is no}\\ \text{ carry or an even carry. Add to the carry the contribution of }W^2~ to ~ E_t.\\\text{This is always even. So this makes}~E_t ~an~ even~digit ! \implies E ~is~ only ~even.\\ \text{So only option remaining O=2 or 8 and W= 2 or 3. We can not have} \\\text{ W=O=2. So only options remaining are 128, 132, and 138.}\\ \text{So we see that }~138^2=19044.. ~~~~~~hits ~the ~target !!!!\\T+W+H+R=1+3+9+0=~~~\boxed{\color{#D61F06}{13}}

0 pending reports

×

Problem Loading...

Note Loading...

Set Loading...